Difference between revisions of "2015 AIME II Problems/Problem 8"

m (Add Solution Section)
Line 4: Line 4:
  
 
==Solution==
 
==Solution==
 +
 +
==See also==
 +
{{AIME box|year=2015|n=II|num-b=7|num-a=9}}
 +
{{MAA Notice}}

Revision as of 10:16, 27 March 2015

Problem

Let $a$ and $b$ be positive integers satisfying $\frac{ab+1}{a+b} < \frac{3}{2}$. The maximum possible value of $\frac{a^3b^3+1}{a^3+b^3}$ is $\frac{p}{q}$, where $p$ and $q$ are relatively prime positive integers. Find $p+q$.

Solution

See also

2015 AIME II (ProblemsAnswer KeyResources)
Preceded by
Problem 7
Followed by
Problem 9
1 2 3 4 5 6 7 8 9 10 11 12 13 14 15
All AIME Problems and Solutions

The problems on this page are copyrighted by the Mathematical Association of America's American Mathematics Competitions. AMC logo.png